You are on page 1of 36

Lecture: Capital Budgeting

Example 1
You are evaluating a new project and you have estimated the following cash flows:

Year 0:

CF = -165,000

Year 1:

CF = 63,120

Year 2:

CF = 70,800

Year 3:

CF = 91,080

Your required return for investments of this risk is 12%.


a. Calculate the NPV and the IRR of this project. Do we accept or reject the
project based on the NPV rule? IRR rule?
b. Payback method: Assume that we will accept the project if it pays back within
two years. Do we accept or reject the project?

Solution:
a. See excel file on blackboard
b.
Year 1: 165,000 63,120 = 101,880 still to recover
Year 2: 101,880 70,800 = 31,080 still to recover
Year 3: 31,080 91,080 = -60,000 project pays back in year 3
The payback period is year 3 if you assume that the cash flows occur at the end of
the year as we do with all of the other decision rules. If we assume that the cash
flows occur evenly throughout the year, then the project pays back in 2.34 years. So
the payback method with 2-year cutoff will result in a rejection of the project.
Example 2
Jordan, Inc. is considering a proposal to manufacture high-protein milkshakes. The
project would use an existing warehouse, which is currently rented by another firm.
The next year's rental charge on the warehouse is $100,000, and will remain at that
level each year. You can assume that the warehouse would be rented out again
starting in year 5.
In addition to using the warehouse, the proposal envisages an investment in plant &
equipment of $1.2 million. This could be depreciated for tax purposes straight-line
to zero over 10 years. However, Jordan, Inc. expects to terminate the project at the
end of four years and to resell the plant and equipment in year 4 for $700,000.
Finally, the project requires an initial investment of net working capital of $350,000.
Thereafter, net working capital is forecast to be 10% of sales in each of years 1
through 4.

Sales of the high-protein milkshakes are expected to be $4 million in each of the


four years of the projects life. Manufacturing costs are expected to be 85% of
sales, and profits are subject to corporate tax of 35%. The appropriate weighted
average cost of capital for the life of this project is 12%. What is the NPV today of
Jordan, Inc.'s proposed idea?
Solution: Excel spreadsheet on blackboard.

Lecture: Valuation (DCF, WACC, Flows to Equity)


Example 1: Spreadsheet on blackboard

Example 2:
Goodyear is thinking of divesting one of the plants. The plant will generate FCF of
$3.8m at the end of the first year and the cash flows will grow at 3% forever. The
plant is financed with a D/V of 0.5 which is expected to remain constant. Goodyear
has an equity cost of capital of 10% and a debt cost of capital of 6% and a marginal
tax rate of 40%. If the plant has an average risk similar to the whole firm, value the
plant using the WACC method.
WACC = Re*E/(E+D) + Rd*(1-T)*D/(E+D) =0.5*0.1 + 0.5*0.06*0.6 = 6.8%
V = FCF1/(WACC - g) = 3.8/(0.068-0.03) = 100
Example 3:
Assume you are analyzing a company with the following cash flows for the next 5
years:

Assume also that cost of equity 13.625% and that the firm can borrow long term at
10%. Tax rate is 50%. Estimate the value of the firm using WACC and the value of
the equity using Flows to Equity valuation. Assume in your estimation market value
of the equity of $1,073 and the value of debt outstanding of $800.
Solution:
Treat these cash flows as given. Note that we are assuming a perpetual bond with
fixed interest payments in each period. The costs of equity and debt are also given.
Note that $1.073 and $800 are not only the market values of debt and equity, but
also the estimated ones, which we use in the calculation of the FCFE above.
V(Equity) = 50/1.13625 + 60/1.13625^2 + 68/1.13625^3 + 76.2/1.13625^6 +
(83.49+1603)/1.13625^5 = 1073
V(wacc) = ?

WACC = Re*(E/(D + E)) + Rd(1-T)(D/(D+E)) = 13.625%(1073/1873) + 5%(800/1873)


= 9.94%
V(firm) = 90/1.0994 + 100/1.0994^2 + 108//1.0994^3 + 116.2/1.0994^4 +
(123.49+2363.01)/1.0994^5
V(Equity) = V(firm) MV(Debt) = 1,073
The reason the two values converge is because the market values of equity and
debt for this firm happen to be equal to the estimated values of equity and debt. If
this does not hold, the weights you use for your cost of capital calculation - which
are based upon current market value - will not be consistent with the debt ratio you
are assuming for estimating for your FCFE calculations and the two values will
diverge.
Example 4:
You are acquiring a target which is in a different industry. The target is currently
financed with a leverage ratio of 0.5. You are planning on financing the acquisition
with a leverage ratio of 1. Your current leverage ratio is 0.75. What is the applicable
discount rate for valuing the target?
Discount Rate

Industry

Leverage

WACC-1

Target Industry

0.5

WACC-2

Target Industry

0.75

WACC-3

Target Industry

WACC-4

Your Industry

0.5

WACC-5

Your Industry

0.75

WACC-6

Your Industry

Solution: WACC-3. This is intuitive you are matching the industry and the capital
structure and will be entirely clear once we cover unlevering/relevering.

Lecture: Cost of Capital

Example 1:
a. Your firm is planning to invest in an automated packaging plant. A comparable firm,
Harburtin Ind. is an all equity firm that specializes in this business. Suppose
Harburtins equity beta is 0.85, Rf= 4%, and the market risk premium is 5%. If your
firms project is all equity financed, estimate its cost of capital.
b. You decided to look for other comparables to reduce estimation error in your cost of
capital estimate. You find a second firm, Thurbinar Design, which is also engaged in
a similar line of business. It has a stock price of $20 per share, with 15 m shares
outstanding. It also has $100m in outstanding debt, with a yield on the debt of
4.5%. Thurbinars equity beta is 1.00
1. Assume Thurbinars debt has a beta of zero. Estimate its unlevered beta. Use
the unlevered beta and the CAPM to estimate its unlevered cost of capital
2. Estimate Thurbinars equity cost of capital using the CAPM. Then assume its
debt cost of capital equals its yield, and using these results, estimate
Thurbinars unlevered cost of capital
3. Explain the difference between your estimate in part (1) and part (2)
4. You decide to average your results in part (1) and part (2), and then average
this result with your estimate from the previous comparable (a). What is your
estimate for the cost of capital of your firms project?
Solution:
a.

p = 0.85 (using all equity comp)


rp

= 4% + 0.85(5%) = 8.25%

b.
1. E = 20 15 = 300
E+D = 400

U = 300/400 1.00 + 100/400 0 = 0.75

rU

= 4% + .75(5%) = 7.75%

Altternatively: Re = 4% + 1*5% = 9%, Rd = Rf (b/c BetaD = 0).


Ru = 9*300/400 + 4*100/400 = 7.75
2.

rE

= 4% + 1.0 5% = 9%

rU

= 300/400 9% + 100/400 4.5% = 7.875%

3. In the first case, we assumed the debt had a beta of zero, so rd = rf = 4%. In the
second case, we assumed rd = ytm = 4.5%
4. Thurbinar

rU

= (7.75 + 7.875)/2 = 7.8125%. Harburtin

rU

= 8.25%

Estimate = (8.25% + 7.8125%)/2 = 8.03%


Example 2:
Unida Systems has 40 million shares outstanding trading for $10 per share. In
addition, Unida has $100 million in outstanding debt. Suppose Unidas equity cost of
capital is 15%, its debt cost of capital is 8%, and the corporate tax rate is 40%.

a.
b.
c.

What is Unidas unlevered cost of capital?


What is Unidas after-tax debt cost of capital?
What is Unidas weighted average cost of capital?

Solution:
a.

What is Unidas unlevered cost of capital?


E = 40 $10 = $400
D = $100

rU

b.

= 400/500 15% + 100/500 8% = 13.6%


What is Unidas after-tax debt cost of capital?

r D =8% (1-40%) = 4.8%

c. What is Unidas weighted average cost of capital?

r WACC

= 400/500 15% + 100/500 4.8% = 12.96%

Example 3
You would like to estimate the weighted average cost of capital for a new airline
business. Based on its industry asset beta, you have already estimated an
unlevered cost of capital for the firm of 9%. However, the new business will be 25%
debt financed, and you anticipate its debt cost of capital will be 6%.
If its corporate tax rate is 40%, what is your estimate of its WACC?
Solution:
We are given pre-tax Rd = 6%. What will be the effective Rd?

rU

= 9% = 75%

rE

= (9% 25%(6%))/75% = 10%

r WACC

rE

+ 25%

rD

= 75%

rE

+ 25%(6%)

= 75%(10%) + 25%(6%)(1 40%) = 8.4%

Example 4
A firm will have EBIT this coming year of $15m. It will also spend $6m on total
capital expenditures and increases in net working capital, and have $3m in
depreciation expenses. This is currently an all-equity firm with a corporate tax rate
of 35% and a cost of capital of 10%.
a. If this firm is expected to grow by 8.5% per year, what is the market value of its
equity today?
b. If the interest rate on its debt is 8%, how much can the firm borrow now and still
have nonnegative net income this coming year?
c. Is there a tax incentive for this firm to choose a D/V ratio that exceeds 50%?
Solution
a.

FCF=EBIT ( 1T ) + DepCapex NWC=15 ( 10.35 ) +36=6.75

E=

6.75
=$ 450 m
10 8.5

b. Interest expense of $15m means debt of 15/0.08 = $187.5m


c. No.
Lecture: Capital Structure and Taxes and Adjusted Present Value (APV)
Example 1: A firm maintains D/E=0.85,

r D=7 , r E =12 , T c =40 , Mkt Cap=$ 220 m .

The FCF is expected to be $10m in one year, and stay constant in perpetuity.
1. What constant expected future growth is consistent with the current
valuation?
2. Estimate the value of DTS
Solution:
1. D/E = 0.85 V = 1.85 E

r WACC =

1
0.85
12 +
7 (10.4 )=8.42
1.85
1.85

V L =E+ D=220 1.85=407=

FCF
10
=
WACC g 0.0842g

g=5.96
2. Pretax WACC:

r pWACC =

V U=

1
0.85
12 +
7 =9.7%
1.85
1.85

FCF
10
=
=$ 267 m
r pWACC g 0.0970.0596

PV[DTS]=407-267=$140m

Example 2: A firm expects FCF=$5m each year.

T c =0.35 . Unlevered cost of

capital is 15%. The firm has outstanding debt of $19.05m, and expects to maintain
this level of debt in perpetuity
1. What is the value of the firm without leverage?

2. What is the value of the firm with leverage?


Solution:

5
=$ 33.33 M
0.15

1.

V U=

2.

V L =V U +T c D=33.33+0.35 19.05=$ 40 m

Example 3: A firm is evaluating a project that will provide an FCF of $18 for 4
years, starting next year, and an FCF of -28m today. The risk of the project is similar
to that of the rest of the firm. The firm has: Cash = $20m, D = $320m, E = 300, Rd
= 6%, Re = 10%. Tc = 40%. The firm will maintain similar D/E ratio in the future.
1. Determine the value of the project using WACC
2. Determine the project debt capacity (amount of D that is required to maintain
the D/V ratio)
3. Determine the value of the project using APV
Solution:
1.
Net D = 320 20.
V = E+D = $600m
WACC = E/(E+D) * Re + D/(E+D) * Rd * (1-T) = 300/600 * 10% + 300/600 * 6% * (10.4) = 6.8%
VL = 18/1.068 + 18/1.068^2 + 18/1.068^3 + 18/1.068^4 = 61.25
NPV = 61.25-28 = $33.25m
2.
Debt Capacity: We are assuming constant D/E ratio. How do we implement this?
The firm has to finance its investments with the same mix of D/E as the rest. So in
this example, it will add 61.25*50%=$30.625M in new D. It can add this D by
borrowing or reducing Cash.
Only 28 is required for the project will pay 30.625-28 = 2.625 to shareholders.

We do the same thing for each period. That is, calculate V L and get the implied level
of debt (D/V)*VL. This will give you a debt value for each year that is needed to
maintain the assumed D/V ratio.
For year 0 we have: Total Assets = 661.25; D = 330.625; E = 330.625 total
Liabilities=661.25
The firm only needed 28 to invest, but it borrowed 30.625. In order to maintain the
constant D/V ratio, the remainder can be distributed to shareholders. The E
increased by NPV-Payout = 33.25-2.625. So they got the full NPV (if you add
dividend).
APV: Ru = 0.5 * 10% + 0.5*6% = 8% Vu = 18/1.08 + 18/1.08^2 +
18/1.08^3+18/1.08^3 = $59.62m
For the remainder of 2 and 3, as well as for the way to calculate APV with and
without implied debt capacity from WACC see the accompanying spreadsheet.

Lecture: Introduction to Capital Structure


Example 1
A firm is financed solely by common stock. It has 3m shares outstanding with a
market price of $12 per share. The CEO announces that the firm intends to issue
$6m of debt, and use the proceeds to repurchase shares
1.

What is the market value after this change in capital structure?

2.

How is the market price of the stock affected by the announcement?

3.

How many shares can the firm repurchase with the $6m of new debt that
they issue?

4.

Who gains or loses?

Solution:
This transaction can be viewed in two stages. First, the firm raises $6m in cash.
Second, it uses the cash to repurchase shares. Let

Pold , Pnew

be old and new share

prices
Let

S old , S new

be old and new number of outstanding shares.

Solve two equations:

S new =

S old R

1.

Pnew = (V-D)/( S old -R) = ($36-$6)/(3-R)

2.

R Pnew

Pnew

= D = $6

= 12 and R = 0.5m

How do we get the firm value after the change for equation 1? We are in MM world,
so the value will be the same.
Both equity and debt holders are equally well off. Why?

Example 2 (Manipulating P/E ratio):


A firm generates expected cash-flow of $5m per year in perpetuity. Market value of
debt is $10m. Debt is rolled over and risk free with

rd

= 4%. The firm has 4m

shares, priced at $10 (V = $50m)


1. Solve for

rA

2. Solve for Earnings per Share (EPS)


3. Solve for Price-Earnings ratio (P/E)
4. Solve for

rE

Solution:
Why is V= $50? PV(D) + PV(E) = $10m + $10*4m = $50m
1.

Solve for

rA

by inverting the perpetuity equation V=C/ r A , so

=5/50=10%
2.

EPS = (revenue interest)/number of shares


= ($5 (4%*$10))/4
= $1.15/share

3. P/E = $10/$1.15 = 8.7


4. Relevering:

r E=10 +

( 1040 )( 10 4 )=11.5

rA

Example 2 (contd)
The CEO decides to boost P/E ratio in order to increase shareholder value. The
firm announces that it will issue new shares in order to buy back all outstanding
debt.
1. How many shares do they need to issue?
2. What happens to the share price?
3. What happens to the EPS, P/E ratio, and

rE ?

Solution:
Let S be the number of new shares issued, and let

Pnew

be the new share price.

Without any debt, E = V = $50 million. Why?


1+2)Again, solve two equations:
1.
2.

Pnew = new value of E/new total # of shares = $50/(4m + S)


S Pnew =$ 10

million

Solving these gives: S = 1 million,

Pnew

= $10 the same as before.

3. EPS decreased to $5/5 = $1/share


P/E goes up to $10/$1 = 10

r E = 10% + (0/50)*(10%-4%)
We can see that an increase in P/E ratio does not mean an increase in the value of
the firm. We used the MM result on the irrelevance of the recap for the firm value.
Example 3 An all-equity firm with assets worth $25b and 10b shares outstanding
plans to borrow $10b and use these funds to repurchase shares. The firms Tc=35%,
and it plans to keep its outstanding debt equal to $10b permanently.
a.

Without the increase in leverage, what would their share price be?

b.

Suppose the firm offers $2.75 per share to repurchase its shares. Would
shareholders sell for this price?

c.

Suppose the firm offers $3.00 per share, and shareholders tender their shares at
this price. What will their share price be after the repurchase?

d.

What is the lowest price the firm can offer and have shareholders tender their
shares? What will its stock price be after the share repurchase in that case?
Solution:

a.

Without the increase in leverage, what would their share price be?
P= 25/10 = $2.5per share

b.

Just before the share repurchase:


Assets = 25 (existing) + 10 (cash) + 35%*10 (tax shield) = $38.5b
E = 38.5 10 = 28.5
P = 28.5/10 = 28.5 per share
Therefore, shareholders will not sell for $2.75 per share

c.

Assets = 25 (existing) + 35%*10 (tax shield) = $28.5b


E = 28.5 10 = 18.5b
Shares = 10 -10/3 = 6.667b
Share price: P = 18.5/6.667 = $2.775

d.

From the second question, fair value of the shares prior to repurchase is $2.85.
At this price the firm will have 10-10/2.85 = 6.49m shares outstanding, which will be
worth 18.5/6.49 = $2.85 after the repurchase. Therefore, shares will sell at this
price

Lecture: Taxes and Low Leverage Puzzle


Example 1: Interest rate tax deduction
Consider a firm which had earnings before interest and taxes of $1.85 billion in
2008, and interest expenses of about $350m. Its marginal corporate tax rate was
35%.

What is this firms Net Income with and without leverage?

What is the total amount available to all investors?

w/leverage

w/o leverage

EBIT

$1850

$1850

Interest Expense

-350

Income before tax

1500

1850

-525

-648

Taxes (35%)
Net Income

$975

$1202

w/leverage

w/o leverage

Interest paid to debt holders

350

Income available to equity holders

975

1202

Total available to all investors


$1202

$1325

Example 3: An all-equity firm with 20m shares outstanding has a stock price of
$7.50. Although investors currently expect the firm to remain an all-equity firm, it
plans to announce that it will borrow $50m and use the funds to repurchase shares.
The firm will pay interest only on this debt, and it has no further plans to change the
amount of debt. Tc = 40%.

What is the market value of their assets before the announcement?

What is the market value of the firms assets (including any tax benefits) just
after the debt is issued, but before the shares are repurchased?

What is their share price just before the repurchase? How many shares will
they repurchase?

What are their market value balance sheet and share price after the
repurchase?

Solution:
1. Assets = Equity=$7.5*20=$150m

2. The question is asking about ASSETS. What does this firm have on the asset side
of its balance sheet at this point? Part of it is equal to equity; thats what the firm
had before. Part of it is the cash we got from D. And part of it is DTS.
Assets = E = 150(existing) + 50(cash from D) + 40%*50(tax shield) = $220m
3. E = Assets Debt = 220 50 = $170 million.
P = $170m/20= $8.5
They will repurchase R = 50/8.5 = 5.882m shares. You can get same values of P and
R if you solve the two equations method. There, we are looking at the price right
after the repurchase, so dont forget to subtract cash.
4. Assets = 150 (existing) + 40% 50 (tax shield) = $170 million
Debt = $50 million; E = A D = 170 50 = $120 million; P= $120/(20-5.882) =
$8.5/share

Lecture: Costs of Financial Distress


Example 1: Which type of firm is more likely to experience a loss of customers in
the event of financial distress?
a. Campbell Soup Company or Intuit, Inc. (a maker of accounting software)?
b. Allstate Corporation (an insurance company) or Reebok International (a footwear
and clothing firm)?
Solution:
a. Intuit Inc.its customers will care about their ability to receive upgrades to their
software.
b. Allstate Corporationits customers rely on the firm being able to pay future
claims.

Example 2: Which type of asset is more likely to be liquidated for close to its full
market value in the event of financial distress:
a. An office building or a brand name?
b. Product inventory or raw materials?
c. Patent rights or engineering know-how?
Solution:
a. Office buildingthere are many alternate users who would be likely to value the
property similarly.
b. Raw materialsthey are easier to reuse.
c. Patent rightsthey would be easier to sell to another firm.

Example 3 Kohwe Corporation plans to issue equity to raise $50m to finance a new
investment. After making the investment, Kohwe expects to earn FCF of $10m each
year. Kohwe currently has 5m shares outstanding, and it has no other assets or
opportunities. Suppose the appropriate discount rate for future FCF is 8%, and the
only capital market imperfections are corporate taxes and financial distress costs.

a. What is the NPV of Kohwe investment?


b. What is Kohwes share price today?
c. Suppose Kohwe borrows the $50m instead. The firm will pay interest only on this
loan each year, and it will maintain an outstanding balance of $50m on the loan.
Corporate tax rate is 40%, and expected FCF are still $10 million each year. What is
Kohwes share price today?
d. Now suppose that with debt, Kohwes expected FCF will be $9 million per year
due to reduced sales and other CFD. Assume that the appropriate discount rate for
Kohwes FCF is still 8%. What is Kohwes share price today given the CFD?
a. 10/0.08 50 = $75m
b. 75/5 = $15/share
c.

75+ .450
=$ 19 / share.
5

d.

9
50+0.450
0.08
=$ 16.5 / share.
5

Lecture: Agency problems and Capital Structure


Example 1: Wealth Transfer
Two periods: t = 0, and t = 1. At t = 1 the state is either boom or bust, with equal
probability. Value of the firm goes to 150 in boom and 50 in bust. Assume:
and

rf = 0

=0 . Firm has debt with face value of 50 due at t = 1.

Now, the firm issues more debt with face value of 50 and equal seniority to the
existing debt (also called pari passu debt), repurchase shares:
1. Value of old debt post-issue?
2. Value of new debt?
3. Value of equity post-issue?
4. Value of equity post-issue plus payment to shareholders?
5. Who gained and who lost?
6. Was value of total claims maintained?
We will use D and E to represent market values. Initially, we have:

V = $150
V = $100

V = $50

After issuing pari passu debt:

V = $150
V = $100

Value of old debt post issue: 75/2 = $37.5 < 50. Lose
V $12.5
= $50

Value of new debt: 37.5. Neither lose nor gain

Value of Equity post-issue: E1 = 25

E1 + payment = 25 + 37.5 = 62.5. The difference $12.5

Old creditors lose. Shareholders gain. The total value of claims is unchanged.

Example 2 (Risk Shifting): Initial setup:

V = $100
V = $60

Prob =
1/2

(D, E) = ($50,

(D,E) = ($35, $25)


V = $20
(D,E) = ($20, 0)
New Investment opportunity. NPV = -2.5:

Payoff = $5
Prob =
1/2

Cost = 0

Payoff = -$10

What is the change in the value of D? E?

Do shareholders want to undertake the project?

How do initial creditors feel about this?

Does MM hold?

With this investment:

V = $105
(D, E) = ($50,

V = $57.5
(D,E) = ($30,
V = $10
(D,E) = ($10, 0)
Debt holders lose $5, but shareholders gain only $2.5. They would like to invest, but
it is not a harmless transfer anymore, as the value of the firm decreases by the
NPV of the project. Therefore, MM does not hold.
Example 3 (Debt Overhang): Initial setup:
V = $100
V = $60
(D, E) = ($35,

V = $20
(D, E) = ($20,
The firm is evaluating a new investment opportunity that costs $10 and returns $15
tomorrow in both states of the world.

What would be the effect of this investment on the values of D and E at t=0?

Can we raise $10 of new equity?

How about issuing new debt?

Solution:

What would be the effect of this investment on the values of D and E at t=0?

We need to check what happens to the value of equity if the firm invests. If the
present value of equity increases by at least the amount needed to invest in the
project, current shareholders will be able to finance this investment with equity.
V = $115

V = $75
(D, E) = ($42.5,

V = $35
(D, E) = ($35,
The most equity holders can gain from this project (in expectation) is 7.5 because in
the bad state the proceeds of the project will go to debt holders. Therefore, the firm
will not be able to finance this project with equity. In other words, in order to raise
$10 in new equity, old shareholders will have to promise new equity a payoff higher
than the proceeds of the project.
How about issuing new D?
Let us consider potential debt issues of different seniority.
1. Junior debt:
A maximum face value that the firm can promise junior debt holders is 15. If it
promises more, the incumbent shareholders will lose money on this project. But
even with this face value, the present value of junior debt is $7.5. This is because in
the bad state of the world the debt is junior to old debt holders, and therefore it is
wiped out, just like equity. Therefore, we cannot finance this project with junior debt.
2. Pari passu (equal seniority) debt:
What face value do we need to promise new debt holders in order to get $10 from
them today? This will help us answer the question of whether it is possible to
finance this project with equal seniority debt. If this face value is higher than $15,
incumbent equity holders will not invest.

The present value of the new debt holders claim is $10 (this is the investment we
need them to finance). It has to be equal to the present value of their claim. In the
good state, new debt holders will get F. In the bad state, they will get 35*F/(50+F).
When the firm doesnt have enough cash to cover its debt obligations, we split the
cashflows between debt holders of equal seniority according to the face values of
their claims. So the old debt holders will get (in the bad state): 35*50/(50+F). This
gives us the following equation:
10= 0.5*F + 0.5*35*F/(50+F). Solving this for F gives F = 12.84589. This is the face
value of new equal seniority debt that we need to promise to our new debt holders.
We can see right away that shareholders will be willing to invest under these terms;
they still keep some of the proceeds of the project in the good state (1512.8459=2.1541), and in the bad state their situation does not change (they get
zero). So, if they invest, the present value of E will increase by 0.5*2.1541=1.07705.
Old debt holders will also want the firm to issue pari passu debt. In the good state
their situation does not change (they get 50), but in the bad state they get 35*50/
(50+12.8459)=27.84589, which is more than what they would get without the
investment ($20). So the present value of the old debt increases by 3.92.
New debt holders are indifferent they will break even on their transaction. As we
saw above, the present value of their claim equals exactly to what they invest
($10).
In fact, in this example, existing debt holders would be better off (relative to no
issuance) even if the firm issues senior debt with the face value of $10. In reality, as
we discussed in class, existing debt contracts often include covenants that forbid
issuing new debt. Even if ex post issuing new debt could be beneficial for old debt
holders, we may have to renegotiate existing contracts in order to issue new debt.

Example 4 (Management v. Shareholders)


Empire Inc. forecast for the coming year is as follows (thousands):
EBIT

$1000

Interest expense

Income before tax

1000

Taxes

-350

Net Income

$650

$200k of earnings will be needed to make NPV>0 investments. Unfortunately,


managers are expected to waste 10% of its net income on needless perks. All
remaining income will be returned to shareholders.
a. By how much would each $1 of interest expense reduce Empires dividend and
share repurchases?
b. What is the increase in the total funds Empire will pay to investors for each $1 of
interest expense?
c. What are the two benefits of debt financing for Empire?
Solution
a.
By how much would each $1 of interest expense reduce Empires dividend and
share repurchases?
Net income will fall by $1 0.65 = $0.65. With interest expense of I, well have
(1000-I)*0.65 vs 1000*0.65 without interest expense. So the difference is I*0.65.
Now, because 10% of net income will be wasted, dividends and share repurchases
will fall by $0.65 (1 .10) = $0.585.
b.
What is the increase in the total funds Empire will pay to investors for each $1
of interest expense?
Pay $1 in interest, give up $0.585 in dividends and share repurchases
Increase of 1 0.585 = $0.415 per $1 of interest.
c.

What are the two benefits of debt financing for Empire?

In addition to tax benefits of leverage, debt financing can benefit Empire by


reducing wasteful investment. So here we have 2 sources of gain: DTS ($0.35) and
Agency Benefit (0.065)

Example 5 (Managerial entrenchment)


A firm generates FCF of $90m per year. Ra= 10%. Tc= 40%. A raider is poised to
take over the firm and finance it with $750m in permanent debt. The raider will
generate the same FCF. The takeover will be successful only if the raider can offer a
premium of 20% over the current value of the firm.

What level of permanent debt will the firm choose, according to the managerial
entrenchment hypothesis?

Solution:
Managerial entrenchment hypothesis says that management may issue suboptimal
amount of debt because debt forces them to work harder (agency benefits of debt),
or if it is a risky debt, it may increase the probability of getting fired. How can the
management deter takeover in this case?
Suppose the current value is X and whatever the raider can possibly produce is Y.
We are given that a raider must pay a premium of 20% over X. If 1.2X > Y, the
raider wont make a bid. So if the current value is already higher than Y/1.2, the
raider wont make a bid.
In this example management can pick the right amount of debt to increase the
value of the firm just enough to deter a takeover. In the real world, they could
increase their effort or decrease wasteful activities to achieve the same goal, but
here the only thing they can do is change D and increase the valuation due to a
higher DTS.
What is the maximum value that the raider may hope to get out of this firm? We are
given that they will use D = 750. Therefore, levered value with Raider = V U + PV
(DTS) = 900 + 40 %(750) = $1.2 billion. This is what the raider can hope to get out
of this firm. (Unlevered Value = 90/0.1 = $900)
What is the minimum V that will prevent the raid? To prevent a successful raid,
current management must have a levered value of at least: $1.2 b/1.2 = $1 billion.
(Suppose the valuation is $1.1b. The raider will have to pay $1.1*1.2 = 1.32 > 1.2)
Thus, the minimum tax shield required to deter a takeover is $1 billion 900 million
= $100 million.
To achieve this DTS and prevent the takeover, the incumbent management will have
to issue 100/0.4 = $250 million in debt.

Lecture: Capital Structure and Asymmetric Information


Selling the family business example
You want to sell your family business. Assumptions about the market:
There are 3 types of such businesses: excellent (worth $9m), good (worth $6m), and
lemons (worth $3m). You believe the fractions of each are 1/3, 1/2, and 1/6,
respectively. You know which type you are selling, but cannot credibly convey this
info to the market.

Which firms will be actually sold and at what price? Will you sell if you own an
excellent business? Good? Lemon?

Solution:
Conjectures:
A: All types of firms are for sale
B: Only Good and Lemons on the market
C: Only Lemons on the market
D: Only Excellent firms are on the market
Conjecture A: The market price will be P = 1/3 * $9m + * $6m + 1/6 * $3m =

$6.5. But owners of excellent businesses will not be willing to sell at this

price. Therefore, this conjecture is not an equilibrium. We need to check


other conjectures.
Conjecture B: P= * 6 + * 3 = 4.5 + 0.75 = 5.25. Now, the owners of
good businesses will not be willing to sell. This conjecture is not an
equilibrium.
Conjecture C: P = 3. This conjecture is an equilibrium; owners of lemons will
be on the market. Everyone else will not participate.
Conjecture D: P = 9. Everyone will have an incentive to participate at this
price. Therefore, this conjecture is not an equilibrium, since it assumes that
only excellent types will be on the market.
The outcome is what we call adverse selection. The selection of market
participants and the goods that are actually traded is inferior; good types
chose to stay out of the market because they cannot credibly convey their
quality to outsiders, and therefore will get an average price for their goods.

Example 1 (Issuing equity under asymmetric information):


Consider a Biotech firm (owned by its management). Approval for its drug Celebroxx
is pending from the FDA. It can invest $100m in a NPV > 0 project (e.g., marketing),
that will benefit all production lines. Must decide now, waiting is not an option.
Approval also increases its ability to sell the other products, and the PV of both
Celebroxx and Non-Celebroxx assets depend on the FDAs decision
Approval (prob =
0.5)

Non-approval(prob =
0.5)

PV of other
Assets

150

50

PV of Investment

120

110

Here is the timeline of this game:

Date 0

Management learns from the FDAs


behavior whether it will approve
Date
the drug or
not.1 It has to issue
shares to finance the investment.
The investment is undertaken after
the share issue.

Date 2

FDA
announces its
decision.

At date 1, management gets a private signal from the FDA and learns with certainty
whether the drug is going to get approved. However, the management cannot
credibly convey this information to the market. The problem is that they have to
decide today whether to invest in the marketing project, and therefore they need to
decide whether to issue equity given the current valuation.

Will the management undertake the project if it has to finance with equity?
Assume (for convenience only) that discount rate = 0

Solution:
In class we introduced the recipe for determining an outcome of such a situation:
Step 1: Make a conjecture about the management actions (issue equity/not issue
equity) depending on their information. In this case, the conjectures will be:

A: Always issue and invest

B: Issue only if FDA will not approve

Step 2: Calculate market prices implied by these actions


Step 3: Verify that given these prices the conjectured actions are optimal
Let us examine the two conjectures separately:
A: Always issue and invest.
In this case, no matter what the firm does, no information is revealed to investors
about the FDA approval. Therefore, at t=1, the markets expected value of the firm
including the new project is a weighted average of the value if Celebroxx is
approved (270) and he value if Celebroxx is not approved (160). PV =
0.5*270+0.5*160 = 215
In return for contributing 100, new shareholders will demand 100/215 = 0.465116
of the firm.
We need to figure out whether the old management will find it worthwhile to issue
and invest given our conjecture. To do that we need to check whether they gain or
lose from investing at the terms that they calculated.
New equity leaves management with [1-(100/215)] = 0.534884 of the PV of the
firm. If management knows that FDA approves:

Eold , post issue =

115
( 150+120 )=144.42 . If they dont invest they get 150. Therefore,
215

they will not invest. New shareholders will pay 100, but will get 270*(100/215) =
125.6, a surplus of 25.6
If management knows that FDA doesnt approve:

Eold , postissue =

115
( 50+110 )= 85.58. If they dont invest, they get 50. So they will
215

invest.
Therefore, if the managers know that the drug will get approved, they will not invest
in the new project, which means that conjecture A is not equilibrium and we have to
examine a different conjecture.
B: Issue and invest only if FDA does not approve
Now issuing shares at t = 1 signals bad news. If firm receives good news at t = 1,
according to this conjecture, it does nothing and this signals good news to investors.
If the bad news arrives, the firm issues equity, but there is no asymmetric
information and no mispricing. The markets valuation is PV = 50+110 = 160.
New investors contribute 100 to a firm that is worth 160, so they demand 100/160
= 62.5% of the firm.
After issuing, the current management is left with [1-100/160] = 60/160 = 37.5% of
the firm. Therefore, the current shareholders keep 37.5%*160 = 60 > 50 ($50 is
what they get if they do not invest). This means that they are better off investing in
the project even though issuance will reveal their type.
In general, we still need to verify that the good types will not have an incentive to
issue given this valuation. The market will think of them as bad types, and they will
have to give 62.5% to new shareholders. Old shareholders will retain (60/160)*270
= 101.25 < 150. So they would not do it
Conjecture B is equilibrium.

Example 4:
WRT Inc. is considering expanding into a new market. The expansion will have the
same business risk as the existing assets. It will require an initial investment of
$50m and will generate perpetual EBIT of $20m per year. After the initial
investment, future CapEx are expected to equal depreciation and no further
additions to NWC are anticipated.

WRTs existing capital structure:

$500 million in equity


$300 million in debt (market values)
10 million equity shares outstanding.

The unlevered cost of capital is 10%, and WRTs debt is risk free with an interest
rate of 4%. The tax rate is 35%.
a. WRT initially proposes to fund the expansion by issuing equity. If investors were
not expecting this expansion, and if they share WRTs view of the expansions
profitability, what will the share price be once the firm announces the expansion
plan?
b. Suppose investors think that the EBIT from WRTs expansion will be only $4
million. What will the share price be in this case? How many shares will the firm
need to issue?
c. Suppose WRT issues equity as in part (b). Shortly after the issue, new
information emerges that convinces investors that management was correct
regarding the cash flows from the expansion. What will the share price be now? Why
does it differ from that found in part (a)?
d. Suppose WRT instead finances the expansion with a $50 million issue of
permanent risk-free debt. What is its new share price once the new information
comes out? Comparing your answer with that in part (c), what are the two
advantages of debt financing in this case?
Solution:
a. NPV of expansion= 20*(0.65/0.1) 50 = $80m
E = (500 + 80)/10 = $58/share
Alternatively, you could solve the two equation method. Let S be the number of
shares the firm will issue:
(500+50+80)/(10+S) = P1
P1S = 50
P1 = 58, S = 0.862
b. NPV of expansion = 4*(0.65/0.1) 50 = -24m
Price = (500-24)/10 = $47.6/share
To finance the investment of $50, the firm will have to issue 50/47.6 = 1.05m new
shares. What is the intuition for this? Recall that the firm issues new equity. So the

equity after issue is E=500+50-24 = 526. New shareholders will demand fraction of
shares f=50/526 =0.095057
In terms of number of new shares (Snew), this fraction corresponds to:
Snew = f*(S_old + Snew) Snew = 0.095057*(10+Snew) Snew = 1.05m

c. Price = (500 + 50 + 80)/11.05 = $57/share. The new projects has the NPV =
80, but it gets divided between the old and the new equity, because new equity
was issued at a discount relative to its true value. The true value is now
revealed, but by now the firm has already issued more new shares than it would
without underpricing, and the share price is now lower than in (a). Old
shareholders lost: (58-57)*10 = 10. New shareholders gain: (57-47.6)*1.05 = 10.
d. Tax shield = 35%(50) = $17.5m
Price = (500+80+17.5)/10 = $59.75/share.
500 + 50+ 80+17.5 = enterprise value. Shareholders gain $2.75. The sources of
this gain are the tax shield ($1.75) and the fact that the firm didnt have to sell
underpriced shares ($1).

Lecture: Security Design


Example 1
Consider a convertible bond with $300 face value. Conversion ratio is 15 shares. 20
shares equity outstanding
1. What is the conversion price?
2. At what firm value will stock have this price?

Solution:
To recap, at firm value of $700 the stock would have spot price = conversion price,
which will trigger the conversion.
1. What is the conversion price?

CP=

$ 300
=$ 20
15 shares

2. At what firm value will stock have this price?

V =D+ E
V =$ 300+ $ 20 20
Therefore:

V =$ 20 35=$ 700
The corresponding plot could be found in the slides

Example 2 (Backdoor equity)


Suppose a firm at date 1 will be worth either 400 or 200, with probabilities 30% and
70%. Management already knows which type they are at date 0, but market doesnt
know until date 1. Additional assumptions:

Costs of financial distress

Preclude taking on D 10%*Value for each type of firm

Cost of $10 if D 5%*Value (e.g., because of increased cost of future


borrowing)

Founding management of the good firm wants to demonstrate firms value to the
market before date 1 and is trying to decide on capital structure that will signal
value to the market now
1. What if they sell equity to the market? Will good types want to sell?
2. What if they issue $25 of D and pay a dividend.

Does this achieve the goal of signaling? Drawback?

3. Suppose firm has 93.5 shares of stock outstanding. Firm raises $25 worth of
convertibles with a conversion ratio of 6.5
Solution:
1. What if they sell equity to the market? Will good types want to sell?
Market values both firms at 0.3*$400+0.7*$200 = $260
Good type doesnt want to sell ($400>$260)
2. What if they issue $25 of D and pay a dividend.

Does this achieve the goal of signaling? Drawback?

This would give bad firm $25/200 = 12.5% debt ratio, so bad firm cannot mimic
Would give the good firm $25/400 = 6.25% debt, so the good firm can do it
This achieves the goal of signaling. Drawback? The problem with regular debt in this
example is that although it achieves the goal of signaling, it leads to an increase in
CFD.
3. Suppose firm has 93.5 shares of stock outstanding. Firm raises $25 worth of
convertibles with a conversion ratio of 6.5
Conversion Ratio: CR = 6.5. This means they get 6.5 shares for their $25
convertibles. NOTE: This is not stated in $1000. It usually is, but doesnt have to be.
Again, Ratio is somewhat misleading term, so be careful with it.
Lets see if this is a credible signal. Face value of convertible is $25, issued at par,
maturity is one year. Note that you are giving the option to acquire 6.5% of the firm
(=100-93.5)
Bad firm knows that it will end up with $25/$200 = 12.5% debt on its balance sheet
when its type is revealed. 12.5% > 10%, so it doesnt do the convertible. This is
because we know that bondholders wont convert.
Good firm will want to issue the convertible. Convertible investors will get 6.5%*400
= $26 if they exercise when the type is revealed. This is greater than the $25 face
value, so they want to exercise. Original equity holders of the firm keep 93.5% of
the firm or $374. So they prefer convertible debt to all other possibilities

Lecture: Raising Capital


Example 1 VCs and Startups

Starware Software was founded to develop software. Initially, the founder invested
$800,000 and received 8 million shares of stock. Starware now needs to raise a
second round of capital, and it has identified a VC. This VC will invest $1 million and
wants to own 20% of the company after the investment is completed.
a. How many shares must the VC receive to end up with 20% of the company?
What is the implied price per share of this funding round?
b. What will the value of the whole firm be after this investment (the post-money
valuation)?
Solution:
a. How many shares must the venture capitalist receive to end up with 20% of the
company? What is the implied price per share of this funding round?
After the funding round, the founders 8 million shares will represent 80% ownership
of the firm. To solve for the new total number of shares (TOTAL):
8,000,000 = .80 S
So S = 10,000,000 shares. The VC will end up with 20% must buy 2 million
shares. Given the investment of $1 million for 2 million shares, the implied price per
share is $0.50.
b. What will the value of the whole firm be after this investment (the post-money
valuation)?
After this investment, there will be 10 million shares outstanding, with a price of
$0.50 per share, so the post-money valuation is $5 million.

Example 2 VCs and LPs


VC firm GSB partners raised $100m of committed capital. Each year over the 10year life of the fund, 2% of this capital will be used to pay GSBs management fee.
GSB will only invest $80m (committed capital less lifetime mgmt fees). At the end of
10 years, the investments made by the fund are worth $400m. GSB also charges
20% carried interest on the profits of the fund (net of mgmt fees).
a. If the $80 million in invested capital is invested immediately and all proceeds
were received at the end of 10 years, what is the IRR of the investments GSB
partners made? That is, compute IRR ignoring all management fees.
b. As an LP, you are more interested in your own IRR, that is the IRR including all
fees paid. Assuming that investors gave GSB partners the full $100 million up
front, what is the IRR for GSBs limited partners (the IRR net of all fees paid).

Solution:
a. Assuming the $80 million in invested capital is invested immediately and all
proceeds were received at the end of 10 years, what is the IRR of the
investments GSB partners made? That is, compute IRR ignoring all management
fees.
IRR solves: NPV = 0 =

So

r=

400
80

400
80
(1+r )10

1 /10

( )

1=17.46

a. As an LP, you are more interested in your own IRR, that is the IRR including all
fees paid. Assuming that investors gave GSB partners the full $100 million up
front, what is the IRR for GSBs limited partners (that is, the IRR net of all fees
paid).
$100m invested. Profit = 400 100 = 300. Carried interest = 20%*300 = $60m
LPs payoff = 400 60 = 340
IRR =

340
100

1/ 10

( )

1=13.02

Example 3
You have an arrangement with your broker to request 1000 shares of all available
IPOs. Suppose that 10% of the time, the IPO is very successful and appreciates by
100% on the first day, 80% of the time it is successful and appreciates by 10%,
and 10% of the time it fails and falls by 15%
a. By what amount does the average IPO appreciate the first day; that is, what is
the average IPO underpricing?
b. Suppose you expect to receive 50 shares when the IPO is very successful, 200
shares when it is successful, and 1000 shares when it fails. Assume the average IPO
price is $15. What is your expected one-day return?
Solution
a. By what amount does the average IPO appreciate the first day; that is, what is
the average IPO underpricing?

.10(100%) + .80(10%) + .10(15%) = 16.5%


b. Suppose you expect to receive 50 shares when the IPO is very successful, 200
shares when it is successful, and 1000 shares when it fails. Assume the average
IPO price is $15. What is your expected one-day return on your IPO investments?
Ave. investment = 0.10(50 15) + 0.80(200 15) + 0.10(1000 15) = $3975
Ave. gain = 0.10(50 15 100%) + 0.80(200 15 10%)+0.10(1000 15 15%)
= $90
Return = 90/3975 = 2.3%

Example 4
Your firm has 10 million shares outstanding, and you are about to issue 5 million
new shares in an IPO. The IPO price has been set at $20 per share, and the
underwriting spread is 7%. The IPO is a big success with investors, and the share
price rises to $50 the first day of trading.
a.

How much did your firm raise from the IPO?

b.

What is the market value of the firm after the IPO?

c. Assume that the post IPO value of your firm is its fair market value. Suppose
your firm could have issued shares directly to investors at their fair market value, in
a perfect market with no underwriting spread and no underpricing. What would the
share price have been in this case, if you raise the same amount as in part (a)?
d. Comparing part (b) and (c), what is the total cost to the firms original investors
due to market imperfections from the IPO?
Solution
a. How much did your firm raise from the IPO?
5m (20 7% 20) = $93 million
b.

What is the market value of the firm after the IPO?

15m 50 = $750 million


c. Assume that the post IPO value of your firm is its fair market value. Suppose
your firm could have issued shares directly to investors at their fair market
value, in a perfect market with no underwriting spread and no underpricing.
What would the share price have been in this case, if you raise the same amount
as in part (a)?

Market value of firm assets absent new cash raised = 750 93 = $657 million.
$657m/(10m original shares) = $65.70 per share
Check: 93m/65.70 = 1.4155m new shares,
$750/11.4155 = $65.7
d. Comparing part (b) and (c), what is the total cost to the firms original
investors due to market imperfections from the IPO?
(65.7 50) 10m = $157 million

Example 5 (Rights issue)


MacKenzie Corp currently has 10 m shares outstanding at a price of $40 per share.
The firm would like to raise money and has announced a rights issue. Every existing
shareholder will be sent one right per share of stock that she owns. The company
plans to require five rights to purchase one share at a price of $40/share.
a.

Assuming the rights issue is successful, how much money will it raise?

b. What will the share price be after the rights issue? (Assume perfect capital
markets)
Suppose instead that the firm changes the plan so that each right gives the holder
the right to purchase one share at $8 per share.
c. How much money will the new plan raise?
d. What will the share price be after the rights issue?
e. Which plan is better for the firms shareholders? Which is more likely to raise the
full amount of capital?
Solution
a. Assuming the rights issue is successful, how much money will it raise?
10m shares/5 40 = $80 million
b. What will the share price be after the rights issue? (Assume perfect capital
markets.)
12m total shares, Value = $400 million + 80 million in new capital = $480
Share price = 480/12 = $40

c. How much money will the new plan raise?


10m $8 = $80 million
d. What will the share price be after the rights issue?
$480/20 = $24 per share
e. Which plan is better for the firms shareholders? Which is more likely to raise the
full amount of capital?
Shareholders do not care. In the first case, each share is worth $40, and exercising
the right has 0 npv, so the total value of a share is $40.
In the second case, the share is worth $24, but the right is worth (24 8) = $16, so
the total value from owning a share is $24 + $16 = $40 per share.
However, the second plan is much more likely to be fully subscribed, because
exercising the right is a good deal. In the first case, shareholders are indifferent
between exercising and not exercising.

You might also like